- Joined
- Apr 2025
- Subscription
- Free
#help
I am having trouble simply breaking down the question.
The conclusion states that mega bookstores have increased sales in best-sellers and decreased sales for their less commercial, literary books.
The support for this is that the portion of their best-seller hardcover books doubles from 7% to around 14% from the full pie of hardcover sales, and the thing with the discount discouraging people from buying other hardcovers.
Does the flaw in this question lie in the fact that they only mention hardcovers? Because I did think about the other type of books, have those sales also decreased? I chose B while drilling, but now that I am trying to understand why I got this wrong, I am unsure if my thought process is the right one.
#help
I see why C is correct, but I cannot understand why A is wrong.
I got this question right, but my thought process was different. I thought that the other reason those paintings were sold at a higher price was the stagnant art market. Would this be an incorrect reasoning? #feedback
This question was kind of like a RRE question. The dilemma here is that we need a resolution that balances freedom to spend their money however they like and with equal access to the public (a fairer system, perhaps).
A: This runs contrary to what the resolution should be. This would just make election campaigning more unequal.
B: This would be a good solution by placing a cap on what the maximum spending should be.
C: This just says that the govt should provide a funds but this does not address equal campaigning.
D: Just like A, this runs contrary
E: This does not resolve the dilemma of unequal spending during campaigning
Help...
I still do not understand why D is the right answer choice. When I read this answer, I realized that the dust rising from Earth's surface may have caused dimness in the Sun. Did I go wrong because I made this assumption about the "raising large amounts of dust" to cause the dimness?
argument:
P: Modeling a computer for reasoning is far easier than modeling a computer based on other cognitive senses (seeing, smelling, etc).
C: our understanding of analytics (reasoning) is better than our senses.
GAP: there is a missing link between modeling computers and our understanding.
I narrowed my answer choices to B and D
B is wrong because from what I understood from premise, it states that our understanding helps model the computers. B goes on to say that the better we understand the computer when performing a task, the better we understand it. I think this is reversed.
D, I definitely missed this while PTing, but upon trying to get myself to understand this question, I see how clearly right it is. UGH
Conclusion: University Hospital can decrease their average length of stay.
Why?
Premise: Edgewater Hospital has an average length of stay of 4 days, while University Hospital has an average length of stay of 6 days. When they treat similar illnesses (lets just say an hairline fracture, it takes similar time to recover in both hospitals, lets just say 2 days).
The assumption that the author made, which leads to the flaw, is that it assumed that both hospitals treat the same illnesses. University Hospital could have more patients with more severe illnesses; hence, if they start kicking them out early, it will affect how they recover.
I got this wrong because I did not see the disconnect between similar illnesses and assumed both hospitals treat the same illnesses.
#Help
That is a very interesting argument. I thought it was a given that the fossils would indicate their age. C just felt so much stronger. If any new fossils of the Dromeosaurs were to be discovered predating that of the earliest bird fossil, then the author's conclusion would fall apart. Why is it that C is not a necessary assumption? The sufficient assumption explanation went over my head.
I think the best way to look at this is by viewing it as a percentage vs number flaw. The gap in the argument was whether the percentage decreasing for standard treatment meant that the total amount spent treating Disease X remained the same or decreased. Answer choice E fills this gap. I wish I had seen this, though!
Premise 1: some read poems and believe they have contradictory ideas, even in great poems
Premise 2: poets of great poems never intend to have contradictory ideas
Conclusion: It is wrong to think that the meaning of the poem is what the author intended.
The assumption: If the reader believes in a certain intention from the poem they read, they will think that intention is the meaning.
A: What different readers agree or disagree with is not important to the argument. The correct AC should connect INTENT with MEANING.
B: it doesn't have to be ONE primary idea. It could be multiple. When negated, "when writing a great poem, he or she does NOT intend to have one primary idea." The conclusion is not affected by this. It does not connect intent to meaning.
C: This one was tricky for me because I did not understand it completely but this is wrong because no where in the argument is it saying that the reader won't agree with what the poets intention were. The argument says that whatever the reader believes the poets intention were (contradictory), they will take it as the meaning, which is wrong.
D: Irrelevant, they do not need to discern every idea.
E: YES...what the reader thinks the poet meant as the intention (contradictory) will be taken as the meaning of the poem. This is the underlying assumption.
OVERALL tough question but we got this!.
Explanation:
Premise- There is record that some major meteor impacts preceded mass extinctions. Many mass extinctions also happen without major meteor impacts .
Records also show that many major meteor impacts have not resulted in mass extinctions.
Conclusion: There is no CONSISTENT causal link between major meteor impacts and mass extinctions.
Assumption- for a consistent causal link to be established, every major meteor impact should be followed by mass extinctions. This is an assumption the author should have had while reaching this conclusion from the premise.
By negating AC A, if not all mass extinctions have to be followed by major meteor impacts, it would be a consistent causal link. See how this would prove that the conclusion is false. This is why A is the right choice.
My analysis: I am not sure if it is the best but I thought I would write it down.
Premise: Many people like to live in places of natural beauty. This leads to many relocating here, and with many people moving there, businesses grow.
Conclusion: A government-mandated environment protection initiative would help the overall economy, even though it affects some local businesses.
The necessary assumption here is that even though this government-mandated environment protection initiative affects some local businesses, this doesn't mean it would also affect the newly relocated businesses. If this was the case, and if we negate A/C E, if the initiative does discourage other relocated businesses, then the conclusion falls apart.
A mistake I made was assuming that MOST regions of Natural Beauty have their economies influenced primarily by local businesses, implying that new businesses would not make up for the affected loss of business by the local ones. It is true that there is a chance it would affect the overall economy and would end up defeating the conclusion if negated. However, this is a big MAYBE. We do not know for sure what kind of effect the new businesses will have.
Even if it is based primarily on local businesses, we have to take the information from the question to be true, which mentions that the government-mandated environment protection initiative helps the overall economy.
I hope this helps. I found this question particularly challenging as I was not able to identify the necessary assumption, but upon reviewing it, this is the best explanation came up with to help myself understand better.
is there a video explanation for this passage? If so, then can you link it.
The reason I eliminated B was because of the "as the critics claimed". I thought the coach was affirming that the critics also called them unprofessional. if that part had not been mentioned, I would have definitely selected B. Wouldn't that show that the critics did call them unprofessional?
#help